- PowerScore Staff
- Posts: 5973
- Joined: Mar 25, 2011
- Sat Apr 07, 2018 7:54 am
#94637
Complete Question Explanation
(The complete setup for this game can be found here: lsat/viewtopic.php?f=296&t=2115)
The correct answer choice is (C)
If P leaves exactly one message, then any of the three distributions are still possible. If P’s message is the fifth, then from the contrapositive of the third rule H cannot leave the first message. This information eliminates answer choice (A).
From the sixth rule, when P leaves a message, then H and L also leave messages, with H > L. Because H cannot leave the first message, the earliest message H could leave is the second, and thus the earliest message that L could leave is the third. Thus, answer choice (D) can be eliminated.
The remaining three answers all concern individuals leaving two messages.
Answer choice (B): if T leaves exactly two messages, under the 2-1-1-1-1-0 distribution, F, P, T, H, and L each leave a message, with P > T and H > L. But, with P leaving the fifth message, and the fifth rule stipulating that all of P’s messages precede all of T’s messages, there is not sufficient room to accommodate both of T’s messages and still conform to the fifth rule. Thus, this answer choice is incorrect.
Answer choice (E): if F leaves the third and fourth messages, under the 2-1-1-1-1-0 distribution, F, P, T, H, and L each leave a message, with P > T and H > L. But, with P leaving the fifth message, T must leave the last message, forcing H to leave the first message and L to leave the second message. Because this violates the third rule, this answer cannot be correct.
Thus, answer choice (C) is correct. L could leave the third and fourth messages.
(The complete setup for this game can be found here: lsat/viewtopic.php?f=296&t=2115)
The correct answer choice is (C)
If P leaves exactly one message, then any of the three distributions are still possible. If P’s message is the fifth, then from the contrapositive of the third rule H cannot leave the first message. This information eliminates answer choice (A).
From the sixth rule, when P leaves a message, then H and L also leave messages, with H > L. Because H cannot leave the first message, the earliest message H could leave is the second, and thus the earliest message that L could leave is the third. Thus, answer choice (D) can be eliminated.
The remaining three answers all concern individuals leaving two messages.
Answer choice (B): if T leaves exactly two messages, under the 2-1-1-1-1-0 distribution, F, P, T, H, and L each leave a message, with P > T and H > L. But, with P leaving the fifth message, and the fifth rule stipulating that all of P’s messages precede all of T’s messages, there is not sufficient room to accommodate both of T’s messages and still conform to the fifth rule. Thus, this answer choice is incorrect.
Answer choice (E): if F leaves the third and fourth messages, under the 2-1-1-1-1-0 distribution, F, P, T, H, and L each leave a message, with P > T and H > L. But, with P leaving the fifth message, T must leave the last message, forcing H to leave the first message and L to leave the second message. Because this violates the third rule, this answer cannot be correct.
Thus, answer choice (C) is correct. L could leave the third and fourth messages.
Dave Killoran
PowerScore Test Preparation
Follow me on X/Twitter at http://twitter.com/DaveKilloran
My LSAT Articles: http://blog.powerscore.com/lsat/author/dave-killoran
PowerScore Podcast: http://www.powerscore.com/lsat/podcast/
PowerScore Test Preparation
Follow me on X/Twitter at http://twitter.com/DaveKilloran
My LSAT Articles: http://blog.powerscore.com/lsat/author/dave-killoran
PowerScore Podcast: http://www.powerscore.com/lsat/podcast/